1
$\begingroup$

Based on some experiments, I find that the following two statements are correct. But I can not prove this. At the same time, I still can not find the counterexmaples.

Let $p(x)=x^{n}+a_{2}x^{n-2}+a_{3}x^{n-3}+\dots+a_{n-1}x+a_{n}$ be a polynomial with interger coefficients, where $a_{k}\geq0$ for every even $k$ and $a_{k}\leq0$ otherwise. Note here that the coefficient of $x^{n-1}$ is equal to 0. Suppose that there exists some odd number $p$ such that $a_{p}<0$ and $a_{p-1}>0$. Then the following two statements should be true:

  1. $p(x)$ has at most one nonzero real root.

  2. $p(x)$ has no pure imaginary zeros, i.e. $p(x)$ has no zero in the form $\alpha\textrm{i}$, where $\alpha\neq0$ and $\textrm{i}^{2}=-1$.

Thanks for your time.

I am sorry for losing a condition that $a_{2}\geq5$.

$\endgroup$
2
  • 2
    $\begingroup$ Any other conditions left out? $\endgroup$ Mar 1, 2011 at 3:34
  • $\begingroup$ @Gerry Myerson,thanks. The conditions needed are all presented. $\endgroup$
    – Shunyi Liu
    Mar 1, 2011 at 3:38

3 Answers 3

7
$\begingroup$

$(x^3-1)(x^3-2)(x^2+a^2)=x^8+a^2x^6-3x^5-3a^2x^3+2x^2+2a^2$ has both two different non-zero real roots and $\pm ai$ as roots.

EDIT polynomial adjusted for adjusted conditions. If $a_2$ needs to be at least $b$, set $a$ so $a^2\geq b$. :)

$\endgroup$
3
  • $\begingroup$ @Sándor Kovács, thanks for your counterexmaple! I am sorry that I have lost the condition that $a_{2}\geq5$ in my original problem. So, if this condition is added, then the counterexample you given is not appropriate. $\endgroup$
    – Shunyi Liu
    Mar 1, 2011 at 3:34
  • 1
    $\begingroup$ check again. :) $\endgroup$ Mar 1, 2011 at 3:47
  • $\begingroup$ @Sándor Kovács, thank you very much for your elegant counterexample. $\endgroup$
    – Shunyi Liu
    Mar 1, 2011 at 4:03
1
$\begingroup$

It is not true that $p(x)$ has at most one nonzero real root. For instance, the polynomial $p(x)=x^5+0.001x^3-100x^2+x-0.001$ has three real roots. For the second claim, the polynomial $p(x)=x^5+x^3-x^2-1$ has roots at $\pm i$.

$\endgroup$
1
  • $\begingroup$ He specified that $p(x)$ is a polynomial in integer coefficients, so the first counterexample will not work. $\endgroup$ Mar 1, 2011 at 2:39
1
$\begingroup$

With the condition $a_2\ge5$ there's still $x^{16}+5x^{14}+x^{12}+x^8-x^5+x^4-x^3+1$ which vanishes at $i$.

$\endgroup$
1
  • $\begingroup$ @Gerry Myerson. Thank you very much for your counterexample. $\endgroup$
    – Shunyi Liu
    Mar 1, 2011 at 3:52

Your Answer

By clicking “Post Your Answer”, you agree to our terms of service and acknowledge you have read our privacy policy.

Not the answer you're looking for? Browse other questions tagged or ask your own question.